Đến nội dung

Hình ảnh

Kĩ thuật đổi biến trong chứng minh bất đẳng thức


  • Please log in to reply
Chủ đề này có 20 trả lời

#1
NTA1907

NTA1907

    Thượng úy

  • Thành viên
  • 1014 Bài viết

A. Lời mở đầu

Lời đầu tiên xin gửi lời chào đến tất cả các thành viên VMF. Mình xin tự giới thiệu mình là Nguyễn Thành An, học sinh lớp 10A trường THPT Hồng Lĩnh, tỉnh Hà Tĩnh. Như ta đã biết, bất đẳng thức là một trong những dạng toán khó trong các kì thi HSG cũng như vào các lớp chuyên toán. Để có được kĩ năng chứng minh bất đẳng thức tốt thì học sinh ngoài thông minh thì cần biết chắt lọc áp dụng nhiều phương pháp và kĩ thuật để giải quyết bài toán. Bài viết này mình sẽ giới thiệu với các bạn kĩ thuật đổi biến để đưa bất đẳng thức về một dạng đơn giản hơn, qua đó chúng ta sẽ khai thác một bài toán gốc thành các bài toán khó hơn và tìm cách giải chúng.

B. Phương pháp

I. Các bất đẳng thức cần nhớ:

1. Bất đẳng thức AM-GM

Với các số không âm $a_{1}, a_{2},..., a_{n}(n\in \mathbb{N^{*}})$. Khi đó ta luôn có:

$a_{1}+a_{2}+...+a_{n}\geq n\sqrt[n]{a_{1}a_{2}...a_{n}}$

Dấu = xảy ra$\Leftrightarrow a_{1}=a_{2}=...=a_{n}$

2. Bất đẳng thức Svac-xơ(hệ quả của Cauchy-Schwarz)

Với các số bất kì $a_{1}, a_{2},..., a_{n}(n\in \mathbb{N^{*}})$ và các số dương $b_{1}, b_{2},...,b_{n}(n\in \mathbb{N^{*}})$ ta luôn có:

$\frac{a_{1}^{2}}{b_{1}}+\frac{a_{2}^{2}}{b_{2}}+...+\frac{a_{n}^{2}}{b_{n}}\geq \frac{(a_{1}+a_{2}+...+a_{n})^{2}}{b_{1}+b_{2}+...+b_{n}}$

Dấu = xảy ra$\Leftrightarrow \frac{a_{1}}{b_{1}}=\frac{a_{2}}{b_{2}}=...=\frac{a_{n}}{b_{n}}$

II. Phương pháp:

Đối với những loại toán có tích của 3 biến $x,y,z$ bằng $k^{3}$ thì ta có 3 cách đổi biến như sau:

1. Đặt $x=\frac{ka}{b},y=\frac{kb}{c},z=\frac{kc}{a}$

2. Đặt $x=\frac{ka^{2}}{bc},y=\frac{kb^{2}}{ca},z=\frac{kc^{2}}{ab}$

3. Đặt $x=\frac{kbc}{a^{2}},y=\frac{kca}{b^{2}},z=\frac{kab}{c^{2}}$

C. Các ví dụ

Để làm quen với cả 3 cách đổi biến ta sẽ đến với các bài toán sau:

 

Bài toán 1: Cho $a,b,c>0$ thoả mãn $abc=1$. CMR:

$\frac{1}{ab+a+2}+\frac{1}{bc+b+2}+\frac{1}{ca+c+2}\leq \frac{3}{4}$

 

Lời giải: 

Vì $abc=1$ nên ta có thể đặt $a=\frac{x}{y},b=\frac{y}{z},c=\frac{z}{x}$($x,y,z>0$)

Khi đó bất đẳng thức cần chứng minh tương đương với:

$\frac{1}{\dfrac{x}{z}+\dfrac{x}{y}+2}+\frac{1}{\dfrac{y}{x}+\dfrac{y}{z}+2}+\frac{1}{\dfrac{z}{y}+\dfrac{z}{x}+2}\leq \frac{3}{4}$

$\Leftrightarrow \frac{yz}{xy+xz+2yz}+\frac{zx}{yz+xy+2zx}+\frac{xy}{zx+yz+2xy}\leq \frac{3}{4}$

Áp dụng bất đẳng thức $\frac{1}{m+n}\leq \frac{1}{4}(\frac{1}{m}+\frac{1}{n})$($m,n>0$) ta có:

$\frac{yz}{xy+zx+2yz}=yz.\frac{1}{(xy+yz)+(zx+yz)}\leq \frac{yz}{4}(\frac{1}{xy+yz}+\frac{1}{xz+yz})=\frac{1}{4}(\frac{z}{x+z}+\frac{y}{x+y})$

Thiết lập các bất đẳng thức tương tự rồi cộng vế theo vế ta được:

$\sum \frac{yz}{xy+xz+2yz}\leq \frac{1}{4}(\frac{z+x}{z+x}+\frac{x+y}{x+y}+\frac{y+z}{y+z})=\frac{3}{4}$(đpcm)

 

Bài toán 2: Cho $x,y,z$ là các số thực khác 1 thoả mãn điều kiện $xyz=1$. CMR:

$\frac{x^{2}}{(x-1)^{2}}+\frac{y^{2}}{(y-1)^{2}}+\frac{z^{2}}{(z-1)^{2}}\geq 1$ (IMO 2008)

 

Lời giải:

Vì $xyz=1$ nên ta có thể đặt $x=\frac{a^{2}}{bc}, y=\frac{b^{2}}{ca}, z=\frac{c^{2}}{ab}$($a,b,c>0; a^{2}\neq bc,b^{2}\neq ca,c^{2}\neq ab$)

Khi đó bất đẳng thức đã cho tương đương với:

$\frac{a^{4}}{(a^{2}-bc)^{2}}+\frac{b^{4}}{(b^{2}-ca)^{2}}+\frac{c^{4}}{(c^{2}-ab)^{2}}\geq 1$

Áp dụng bất đẳng thức Svac-xơ ta có:

$\sum \frac{a^{4}}{(a^{2}-bc)^{2}}\geq \frac{(a^{2}+b^{2}+c^{2})^{2}}{(a^{2}-bc)^{2}+(b^{2}-ca)^{2}+(c^{2}-ab)^{2}}$

Ta chứng minh:

$(a^{2}+b^{2}+c^{2})^{2}\geq (a^{2}-bc)^{2}+(b^{2}-ca)^{2}+(c^{2}-ab)^{2}$

$\Leftrightarrow (ab+bc+ca)^{2}\geq 0$(luôn đúng)

$\Rightarrow$ đpcm

Dấu = xảy ra$\Leftrightarrow a=1, b=2, c=\frac{-2}{3} \Rightarrow x=\frac{-3}{4}, y=-6, z=\frac{2}{9}$(TM)

 

Bài toán 3: Cho $x,y,z$ là các số thực khác 1 thoả mãn $xyz=8$. CMR:

$\frac{1}{(x+1)^{2}}+\frac{1}{(y+1)^{2}}+\frac{1}{(z+1)^{2}}\geq \frac{1}{3}$

 

Lời giải:

Vì $xyz=2^{3}$ nên ta có thể đặt $x=\frac{2bc}{a^{2}}, y=\frac{2ca}{b^{2}}, z=\frac{2ab}{c^{2}}(a,b,c>0; a^{2}\neq 2bc,b^{2}\neq 2ca,c^{2}\neq 2ab)$

Khi đó bất đẳng thức cần chứng minh tương đương với:

$\frac{1}{\left ( \dfrac{2bc}{a^{2}}+1 \right )^{2}}+\frac{1}{\left ( \dfrac{2ca}{b^{2}}+1 \right )^{2}}+\frac{1}{\left ( \dfrac{2ab}{c^{2}}+1 \right )^{2}}\geq \frac{1}{3}$

$\Leftrightarrow \frac{a^{4}}{(a^{2}+bc)^{2}}+\frac{b^{4}}{(b^{2}+ca)^{2}}+\frac{c^{4}}{(c^{2}+ab)^{2}}\geq \frac{1}{3}$

Áp dụng bất đẳng thức Svac-xơ ta có:

$\sum \frac{a^{4}}{(a^{2}+bc)^{2}}\geq \frac{(a^{2}+b^{2}+c^{2})^{2}}{(a^{2}+2bc)^{2}+(b^{2}+2ca)^{2}+(c^{2}+2ab)^{2}}$

Ta chứng minh:

$\frac{(a^{2}+b^{2}+c^{2})^{2}}{(a^{2}+2bc)^{2}+(b^{2}+2ca)^{2}+(c^{2}+2ab)^{2}}\geq \frac{1}{3}$

$\Leftrightarrow 3(a^{4}+b^{4}+c^{4})+6(a^{2}b^{2}+b^{2}c^{2}+c^{2}a^{2})\geq a^{4}+b^{4}+c^{4}+4(a^{2}b^{2}+b^{2}c^{2}+c^{2}a^{2})+4abc(a+b+c)$

$\Leftrightarrow a^{4}+b^{4}+c^{4}+a^{2}b^{2}+b^{2}c^{2}+c^{2}a^{2}\geq 2abc(a+b+c)$

$\Leftrightarrow (a^{2}-bc)^{2}+(b^{2}-ca)^{2}+(c^{2}-ab)^{2}\geq 0$(luôn đúng)

$\Rightarrow$ đpcm

Dấu = xảy ra$\Leftrightarrow a=b=c\Rightarrow x=y=z=2$

 

Trên đây là 3 bài toán cơ bản vận dụng cách đổi biến. Để hiểu rõ hơn về cách đổi biến trong các bài bất đẳng thức ta sẽ đến với các bài toán nâng cao. Mở đầu sẽ là một bài toán gốc:

Bài toán 4: Cho $a,b,c>0$. Chứng minh:

$\left ( \frac{a}{a+b} \right )^{2}+\left ( \frac{b}{b+c} \right )^{2}+\left ( \frac{c}{c+a} \right )^{2}\geq \frac{3}{4}$

 

Nhận xét: Khi gặp bài toán này học sinh thường mắc sai lầm sau:

$\sum \left ( \frac{a}{a+b} \right )^{2}\geq \frac{1}{3}\left ( \sum \frac{a}{a+b} \right )^{2}\geq \frac{1}{3}.\left ( \frac{3}{2} \right )^{2}=\frac{3}{4}$

Cách làm trên hoàn toàn sai vì $\sum \frac{a}{a+b}$ không lớn hơn $\frac{3}{2}$ với $a,b,c>0$

Lời giải: 

Bất đẳng thức cần chứng minh tương đương với:

$\Leftrightarrow \sum \left ( \frac{1}{1+\frac{b}{a}} \right )^{2}\geq \frac{3}{4}$

Đặt $\frac{b}{a}=\frac{yz}{x^{2}}, \frac{c}{b}=\frac{zx}{y^{2}}, \frac{a}{c}=\frac{xy}{z^{2}}(x,y,z>0)$

Khi đó ta chứng minh:

$\sum \frac{x^{4}}{(x^{2}+yz)^{2}}\geq \frac{3}{4}$

Áp dụng bất đẳng thức Svac-xơ ta có:

$\sum \frac{x^{4}}{(x^{2}+yz)^{2}}\geq \frac{(x^{2}+y^{2}+z^{2})^{2}}{(x^{2}+yz)^{2}+(y^{2}+zx)^{2}+(z^{2}+xy)^{2}}$

Ta chứng minh:

$\frac{(x^{2}+y^{2}+z^{2})^{2}}{(x^{2}+yz)^{2}+(y^{2}+zx)^{2}+(z^{2}+xy)^{2}}\geq \frac{3}{4}$

$\Leftrightarrow x^{4}+y^{4}+z^{4}+5x^{2}y^{2}+5y^{2}z^{2}+5z^{2}x^{2}\geq 6xyz(x+y+z)$

Mặt khác theo AM-GM ta có:

$VT\geq 6(x^{2}y^{2}+y^{2}z^{2}+z^{2}x^{2})\geq 6xyz(x+y+z)=VP$(đpcm)

Dấu = xảy ra$\Leftrightarrow x=y=z\Leftrightarrow a=b=c$

 

Từ bài tập trên, chúng ta có thể thay đổi hệ số cho các biến a,b,c để có một bài toán khác

Bài toán 5: Cho $a,b,c>0$. Chứng minh:

$\left ( \frac{a}{a+2b} \right )^{2}+\left ( \frac{b}{b+4c} \right )^{2}+\left ( \frac{c}{c+8a} \right )^{2}\geq \frac{3}{25}$

 

Lời giải:

Bất đẳng thức tương đương với:

$\left ( \frac{1}{1+\dfrac{2b}{a}} \right )^{2}+\left ( \frac{1}{1+\dfrac{4c}{b}} \right )^{2}+\left ( \frac{1}{1+\dfrac{8a}{c}} \right )^{2}\geq \frac{3}{25}$

Vì $\frac{2b}{a}.\frac{4c}{b}.\frac{8a}{c}=4^{3}$ nên ta có thể đặt $\frac{2b}{a}=\frac{4yz}{x^{2}},\frac{4c}{b}=\frac{4xz}{y^{2}},\frac{8a}{c}=\frac{4xy}{z^{2}}(x,y,z>0)$

Khi đó bất đẳng thức cần chứng minh tương đương với:

$\frac{x^{4}}{(x^{2}+4yz)^{2}}+\frac{y^{4}}{(y^{2}+4zx)^{2}}+\frac{z^{4}}{(z^{2}+4xy)^{2}}\geq \frac{3}{25}$

Áp dụng bất đẳng thức Svac-xơ ta có:

$\sum \frac{x^{4}}{(x^{2}+4yz)^{2}}\geq \frac{(x^{2}+y^{2}+z^{2})^{2}}{(x^{2}+4yz)^{2}+(y^{2}+4zx)^{2}+(z^{2}+4xy)^{2}}$

Ta cần chứng minh:

$\frac{(x^{2}+y^{2}+z^{2})^{2}}{(x^{2}+4yz)^{2}+(y^{2}+4zx)^{2}+(z^{2}+4xy)^{2}}\geq \frac{3}{25}$

$\Leftrightarrow 22(x^{4}+y^{4}+z^{4})+2(x^{2}y^{2}+y^{2}z^{2}+z^{2}x^{2})\geq 24xyz(x+y+z)$

Chứng minh tương tự như bài toán 4 ta có đpcm

Dấu = xảy ra$\Leftrightarrow x=y=z\Rightarrow 2a=b=c$

 

Với cách khai thác như trên ta hoàn toàn có thể chứng minh được bài toán tổng quát

Bài toán 6: Cho $x,y,z$ là các số thực dương thay đổi và 6 số thực dương cho trước $a,b,c,d,e,g$. CMR:

$\left ( \frac{ax}{ax+by} \right )^{2}+\left ( \frac{cy}{cy+dz} \right )^{2}+\left ( \frac{ez}{ez+gx} \right )^{2}\geq \frac{3}{(m+1)^{2}}$ trong đó $m=\sqrt[3]{\frac{bdg}{ace}}\geq 1$

 

Lời giải:

Bất đẳng thức tương đương với:

$\left ( \frac{1}{1+\dfrac{by}{ax}} \right )^{2}+\left ( \frac{1}{1+\dfrac{dz}{cy}} \right )^{2}+\left ( \frac{1}{1+\dfrac{gx}{ez}} \right )^{2}\geq \frac{3}{(m+1)^{2}}$

Vì $\frac{by}{ax}.\frac{dz}{cy}.\frac{gx}{ez}=\frac{bdg}{ace}=m^{3}$ nên ta có thể đặt $\frac{by}{ax}=\frac{my_{1}z_{1}}{x_{1}^{2}},\frac{dz}{cy}=\frac{mz_{1}x_{1}}{y_{1}^{2}},\frac{gx}{ez}=\frac{mx_{1}y_{1}}{z_{1}^{2}}(x_{1},y_{1},z_{1}>0)$

Khi đó bất đẳng thức cần chứng minh tương đương với:

$\sum \frac{x_{1}^{4}}{(x_{1}^{2}+my_{1}z_{1})^{2}}\geq \frac{3}{(m+1)^{2}}$

Áp dụng bất đẳng thức Svac-xơ ta có:

$\sum \frac{x_{1}^{4}}{(x_{1}^{2}+my_{1}z_{1})^{2}}\geq \frac{(x_{1}+y_{1}+z_{1})^{2}}{\sum (x_{1}^{2}+my_{1}z_{1})^{2}}$

Ta chứng minh:

$\frac{(x_{1}+y_{1}+z_{1})^{2}}{\sum (x_{1}^{2}+my_{1}z_{1})^{2}}\geq \frac{3}{(m+1)^{2}}$

$\Leftrightarrow (m^{2}+2m-2)(x_{1}^{4}+y_{1}^{4}+z_{1}^{4})+(4m+2)(x_{1}^{2}y_{1}^{2}+y_{1}^{2}z_{1}^{2}+z_{1}^{2}x_{1}^{2})\geq m^{2}(x_{1}^{2}y_{1}^{2}+y_{1}^{2}z_{1}^{2}+z_{1}^{2}x_{1}^{2})+6mx_{1}y_{1}z_{1}(x_{1}+y_{1}+z_{1})$

Vì $m\geq 1$ nên đặt $m=n+1(n\geq 0)$.

Khi đó: $(m^{2}+2m-2)(x_{1}^{4}+y_{1}^{4}+z_{1}^{4})+(4m+2)(x_{1}^{2}y_{1}^{2}+y_{1}^{2}z_{1}^{2}+z_{1}^{2}x_{1}^{2})=(n^{2}+4n+1)(x_{1}^{4}+y_{1}^{4}+z_{1}^{4})+(4n+6)(x_{1}^{2}y_{1}^{2}+y_{1}^{2}z_{1}^{2}+z_{1}^{2}x_{1}^{2})\geq (n+1)^{2}(x_{1}^{2}y_{1}^{2}+y_{1}^{2}z_{1}^{2}+z_{1}^{2}x_{1}^{2})+6(n+1)(x_{1}^{2}y_{1}^{2}+y_{1}^{2}z_{1}^{2}+z_{1}^{2}x_{1}^{2})\geq (n+1)^{2}(x_{1}^{2}y_{1}^{2}+y_{1}^{2}z_{1}^{2}+z_{1}^{2}x_{1}^{2})+6(n+1)x_{1}y_{1}z_{1}(x_{1}+y_{1}+z_{1})=m^{2}(x_{1}^{2}y_{1}^{2}+y_{1}^{2}z_{1}^{2}+z_{1}^{2}x_{1}^{2})+6m(x_{1}y_{1}z_{1}(x_{1}+y_{1}+z_{1})$(đpcm)

 

Từ bài toán 4 ta có thể mở rộng bài toán bằng cách thêm biến

Bài toán 7: Cho $a,b,c,d>0$. CMR:

$\left ( \frac{a}{a+b} \right )^{2}+\left ( \frac{b}{b+c} \right )^{2}+\left ( \frac{c}{c+d} \right )^{2}+\left ( \frac{d}{d+a} \right )^{2}\geq 1$

 

Lời giải:

Bất đẳng thức tương đương với:

$\left ( \frac{1}{1+\dfrac{b}{a}} \right )^{2}+\left ( \frac{1}{1+\dfrac{c}{b}} \right )^{2}+\left ( \frac{1}{1+\dfrac{d}{c}} \right )^{2}+\left ( \frac{1}{1+\dfrac{a}{d}} \right )^{2}\geq 1$

Đặt $\frac{b}{a}=\frac{yz}{x^{2}},\frac{c}{b}=\frac{zt}{y^{2}},\frac{d}{c}=\frac{tx}{z^{2}},\frac{a}{d}=\frac{xy}{t^{2}}(x,y,z,t>0)$

Khi đó bất đẳng thức cần chứng minh tương đương:

$\sum \frac{x^{4}}{(x^{2}+yz)^{2}}\geq 1$

Áp dụng bất đẳng thức Svac-xơ ta có:

$\sum \frac{x^{4}}{(x^{2}+yz)^{2}}\geq \frac{(x^{2}+y^{2}+z^{2}+t^{2})^{2}}{\sum (x^{2}+yz)^{2}}$

Ta chứng minh:

$\frac{(x^{2}+y^{2}+z^{2}+t^{2})^{2}}{\sum (x^{2}+yz)^{2}}\geq 1$

$\Leftrightarrow x^{2}y^{2}+y^{2}z^{2}+z^{2}t^{2}+t^{2}x^{2}+2x^{2}z^{2}+2y^{2}t^{2}\geq 2(x^{2}yz+y^{2}zt+z^{2}tx+t^{2}xy)$

$\Leftrightarrow x^{2}(y^{2}+z^{2})+y^{2}(z^{2}+t^{2})+z^{2}(t^{2}+x^{2})+t^{2}(x^{2}+y^{2})\geq 2(x^{2}yz+y^{2}zt+z^{2}tx+t^{2}xy)$(luôn đúng theo AM-GM)

$\Rightarrow$ đpcm

Dấu = xảy ra$\Leftrightarrow x=y=z=t\Rightarrow a=b=c=d$

 

Hướng khai thác thứ 3 của bài toán 4 là nâng luỹ thừa bậc cao hơn

Bài toán 8: Cho $a,b,c>0$. CMR:

$\left ( \frac{a}{a+b} \right )^{3}+\left ( \frac{b}{b+c} \right )^{3}+\left ( \frac{c}{c+a} \right )^{3}\geq \frac{3}{8}$(Đề thi chọn đội tuyển HSG quốc gia Hà Tĩnh)

 

Lời giải:

Cách 1: Với cách đặt như bài toán 4 ta đưa bất đẳng thức vần chứng minh tương đương với:

$\sum \frac{x^{6}}{(x^{2}+yz)^{3}}\geq \frac{3}{8}$

Áp dụng bất đẳng thức Svac-xơ ta có:

$\sum \frac{x^{6}}{(x^{2}+yz)^{3}}\geq \frac{(x^{3}+y^{3}+z^{3})^{2}}{\sum (x^{2}+yz)^{3}}$

Ta chứng minh:

$\frac{(x^{3}+y^{3}+z^{3})^{2}}{\sum (x^{2}+yz)^{3}}\geq \frac{3}{8}$

$\Leftrightarrow 5(x^{6}+y^{6}+z^{6})+13(x^{3}y^{3}+y^{3}z^{3}+z^{3}x^{3})\geq 27x^{2}y^{2}z^{2}+9xyz(x^{3}+y^{3}+z^{3})$

Áp dụng AM-GM ta có:

$3(x^{6}+x^{3}y^{3}+x^{3}z^{3})\geq 9x^{4}yz$

$3(y^{6}+y^{3}x^{3}+z^{3}z^{3})\geq 9xy^{4}z$

$3(z^{6}+z^{3}x^{3}+z^{3}y^{3})\geq 9xyz^{4}$

$2(x^{6}+y^{6}+z^{6})+7(x^{3}y^{3}+y^{3}z^{3}+z^{3}x^{3})\geq 27\sqrt[27]{(xyz)^{54}}=27x^{2}y^{2}z^{2}$

Cộng 4 bất đẳng thức cùng chiều trên ta được đpcm

Cách 2: Theo AM-GM ta có:

$2\left ( \frac{a}{a+b} \right )^{3}+\frac{1}{8}\geq \frac{3}{2}\left ( \frac{a}{a+b} \right )^{2}$

Thiết lập các bất đẳng thức tương tự rồi cộng vế theo vế ta được:

$2\sum \left ( \frac{a}{a+b} \right )^{3}+\frac{3}{8}\geq \frac{3}{2}\left ( \sum (\frac{a}{a+b})^{2} \right )$

Áp dụng bài toán 4 ta có đpcm

Dấu = xảy ra$\Leftrightarrow a=b=c$

 

Để tăng độ khó cho bài tập ta sẽ làm quen với bài toán sau

Bài toán 9: Cho $a,b,c>0$. Tìm GTLN của biểu thức:

$A=\frac{b}{a+3b}+\frac{c}{b+3c}+\frac{a}{c+3a}+abc\left [ \frac{1}{c(a+3b)^{2}}+\frac{1}{b(c+3a)^{2}}+\frac{1}{a(b+3c)^{2}} \right ]$

 

Lời giải:

Ta có:

$3A=3\left [ \frac{b}{a+3b}+\frac{c}{b+3c}+\frac{a}{c+3a}+abc\left ( \frac{1}{c(a+3b)^{2}}+\frac{1}{b(c+3a)^{2}}+\frac{1}{a(b+3c)^{2}} \right ) \right ]=\frac{3b(a+3b)+3ab}{(a+3b)^{2}}+\frac{3c(b+3c)+3bc}{(b+3c)^{2}}+\frac{3a(c+3a)+3ca}{(c+3a)^{2}}=\frac{(a+3b)^{2}-a^{2}}{(a+3b)^{2}}+\frac{(b+3c)^{2}-b^{2}}{(b+3c)^{2}}+\frac{(c+3a)^{2}-c^{2}}{(c+3a)^{2}}=3-\left [ \left ( \frac{a}{a+3b} \right )^{2}+\left ( \frac{b}{b+3c} \right )^{2}+\left ( \frac{c}{c+3a} \right )^{2} \right ]$

Chứng minh tương tự như trên ta cũng có:

$\sum \left ( \frac{a}{a+3b} \right )^{2}\geq \frac{3}{16}$

Do đó $3A\leq 3-\frac{3}{16}\Rightarrow A\leq \frac{15}{16}$

Dấu = xảy ra$\Leftrightarrow a=b=c$

 

----Hết----


Vũ trụ không có biên trong không gian, không có bắt đầu và kết thúc trong thời gian và chẳng có việc gì cho đấng sáng thế phải làm ở đây cả.

 


#2
NTA1907

NTA1907

    Thượng úy

  • Thành viên
  • 1014 Bài viết

Để kết thúc chuyên đề các bạn hãy giải các bài tập sau:

Bài 1: Cho $a,b,c>0$. Chứng minh:

$\left ( \frac{a}{a+2b} \right )^{3}+\left ( \frac{b}{b+2c} \right )^{3}+\left ( \frac{c}{c+2a} \right )^{3}\geq \frac{1}{9}$

Bài 2: Cho $a,b,c>0$. Tìm GTNN của biểu thức:

$\left ( \frac{a}{a+2b} \right )^{4}+\left ( \frac{b}{b+8c} \right )^{4}+\left ( \frac{c}{c+16a} \right )^{4}$

Bài 3: Cho $a,b,c>0$. CMR:

$\frac{a^{2}}{a^{2}+ab+b^{2}}+\frac{b^{2}}{b^{2}+bc+c^{2}}+\frac{c^{2}}{c^{2}+ca+a^{2}}\geq 1$

Bài 4: Cho 3 số thực dương $x,y,z$. CMR:

$\frac{(x+y)^{2}}{x^{2}+y^{2}+2z^{2}}+\frac{(y+z)^{2}}{y^{2}+z^{2}+2x^{2}}+\frac{(z+x)^{2}}{z^{2}+x^{2}+2y^{2}}\leq 3$


Bài viết đã được chỉnh sửa nội dung bởi NTA1907: 16-04-2016 - 16:00

Vũ trụ không có biên trong không gian, không có bắt đầu và kết thúc trong thời gian và chẳng có việc gì cho đấng sáng thế phải làm ở đây cả.

 


#3
PlanBbyFESN

PlanBbyFESN

    Thiếu úy

  • Điều hành viên OLYMPIC
  • 637 Bài viết

Bài 4: Cho 3 số thực dương $x,y,z$. CMR:

$\frac{(x+y)^{2}}{x^{2}+y^{2}+2z^{2}}+\frac{(y+z)^{2}}{y^{2}+z^{2}+2x^{2}}+\frac{(z+x)^{2}}{z^{2}+x^{2}+2y^{2}}\geq 3$

 

Edit :

 

Bài 4:   Cho 3 số thực dương $x,y,z$. CMR:

 

$\frac{(x+y)^{2}}{x^{2}+y^{2}+2z^{2}}+\frac{(y+z)^{2}}{y^{2}+z^{2}+2x^{2}}+\frac{(z+x)^{2}}{z^{2}+x^{2}+2y^{2}}\leq 3$


:huh:


#4
tpdtthltvp

tpdtthltvp

    Trung úy

  • Điều hành viên THCS
  • 831 Bài viết

Để kết thúc chuyên đề các bạn hãy giải các bài tập sau:

 

Bài 3: Cho $a,b,c>0$. CMR:

$\frac{a^{2}}{a^{2}+ab+b^{2}}+\frac{b^{2}}{b^{2}+bc+c^{2}}+\frac{c^{2}}{c^{2}+ca+a^{2}}\geq 1$

 

 

Ta có:

$$\sum \frac{a^{2}}{a^{2}+ab+b^{2}}\geq 1\Leftrightarrow \sum \frac{1}{1+\frac{b}{a}+(\frac{b}{a})^2}$$

Đặt $\frac{b}{a}=\frac{yz}{x^2};\frac{c}{b}=\frac{xz}{y^2};\frac{a}{c}=\frac{xy}{z^2}$

 

Thì BĐT cần CM tương đương:

$$\sum \frac{1}{1+\frac{yz}{x^2}+(\frac{yz}{x^2})^2}\geq 1\Leftrightarrow \sum \frac{x^4}{x^4+x^2yz+y^2z^2}\geq 1$$

Áp dụng BĐT $Schwarz$, ta có:

$$\sum \frac{x^4}{x^4+x^2yz+y^2z^2}\geq \frac{(\sum x^2)^2}{(\sum x^4)+xyz(x+y+z)+(\sum x^2y^2)}\geq \frac{(\sum x^2)^2}{(\sum x^4)+\sum x^2y^2+\sum x^2y^2}=1$$

Vậy ta có đpcm.


$\color{red}{\mathrm{\text{How I wish I could recollect, of circle roud}}}$

$\color{red}{\mathrm{\text{The exact relation Archimede unwound ! }}}$

 


#5
CaptainCuong

CaptainCuong

    Thượng sĩ

  • Thành viên
  • 212 Bài viết

Bài 4: Cho 3 số thực dương $x,y,z$. CMR:

$\frac{(x+y)^{2}}{x^{2}+y^{2}+2z^{2}}+\frac{(y+z)^{2}}{y^{2}+z^{2}+2x^{2}}+\frac{(z+x)^{2}}{z^{2}+x^{2}+2y^{2}}\leq 3$

Ta có

$(x+y)^{2}\leq 2(x^{2}+y^{2})\Rightarrow$ $VT\leq \sum \frac{2(x^2+y^2)}{x^2+y^2+2z^2}$

Đặt $x^2+y^2=a;y^2+z^2=b;z^2+x^2=c$

BĐT cần C/m $\Leftrightarrow \frac{a}{b+c}+\frac{b}{a+c}+\frac{c}{a+b}\leq \frac{3}{2}$(BĐT Nesbit)


Bài viết đã được chỉnh sửa nội dung bởi CaptainCuong: 16-04-2016 - 23:04


#6
anhtukhon1

anhtukhon1

    Sĩ quan

  • Thành viên
  • 480 Bài viết

$\Leftrightarrow \frac{a}{b+c}+\frac{b}{a+c}+\frac{c}{a+b}\leq \frac{3}{2}$(BĐT Nesbit)

wtf? Nesbit nó có phải dấu kia đâu? 


#7
kuhaza

kuhaza

    Trung sĩ

  • Thành viên
  • 102 Bài viết

Ta có

$(x+y)^{2}\leq 2(x^{2}+y^{2})\Rightarrow$ $VT\leq \sum \frac{2(x^2+y^2)}{x^2+y^2+2z^2}$

Đặt $x^2+y^2=a;y^2+z^2=b;z^2+x^2=c$

BĐT cần C/m $\Leftrightarrow \frac{a}{b+c}+\frac{b}{a+c}+\frac{c}{a+b}\leq \frac{3}{2}$(BĐT Nesbit)

BĐT Nesbit ngược lại mak?



#8
PlanBbyFESN

PlanBbyFESN

    Thiếu úy

  • Điều hành viên OLYMPIC
  • 637 Bài viết

Bài 1 và Bài 2 trông có vẻ tương tự mấy bài trên.

 

Bài 3 giải như tpdtthltvp là chuẩn rồi!

 

Bài 4 theo nhận xét của em là dễ, tất nhiên là không dùng đổi biến@


:huh:


#9
NTA1907

NTA1907

    Thượng úy

  • Thành viên
  • 1014 Bài viết

Bài 1 và Bài 2 trông có vẻ tương tự mấy bài trên.

 

Bài 3 giải như tpdtthltvp là chuẩn rồi!

 

Bài 4 theo nhận xét của em là dễ, tất nhiên là không dùng đổi biến@

Nếu bài 1 và 2 em làm được rồi thì post lên để mọi người tham khảo, bài 4 nếu dùng đổi biến thì không dễ đâu...


Vũ trụ không có biên trong không gian, không có bắt đầu và kết thúc trong thời gian và chẳng có việc gì cho đấng sáng thế phải làm ở đây cả.

 


#10
PlanBbyFESN

PlanBbyFESN

    Thiếu úy

  • Điều hành viên OLYMPIC
  • 637 Bài viết

Nếu bài 1 và 2 em làm được rồi thì post lên để mọi người tham khảo, bài 4 nếu dùng đổi biến thì không dễ đâu...

 

Thông cảm em đang ôn thi HK chỉ lướt qua đôi bài vậy thôi, đổi biến này hơi mất thời gian.

 

Bài 4: Lời giải bằng Cauchy-Schwarz rất ngắn gọn:

 

Để kết thúc chuyên đề các bạn hãy giải các bài tập sau:

Bài 4: Cho 3 số thực dương $x,y,z$. CMR:

$\frac{(x+y)^{2}}{x^{2}+y^{2}+2z^{2}}+\frac{(y+z)^{2}}{y^{2}+z^{2}+2x^{2}}+\frac{(z+x)^{2}}{z^{2}+x^{2}+2y^{2}}\leq 3$

 

$\sum \frac{(x+y)^{2}}{x^{2}+y^{2}+2z^{2}}\leq \sum (\frac{x^{2}}{x^{2}+z^{2}}+\frac{y^{2}}{y^{2}+z^{2}})=3$


:huh:


#11
haichau0401

haichau0401

    Thượng sĩ

  • Thành viên
  • 214 Bài viết

Để kết thúc chuyên đề các bạn hãy giải các bài tập sau:

Bài 1: Cho $a,b,c>0$. Chứng minh:

$\left ( \frac{a}{a+2b} \right )^{3}+\left ( \frac{b}{b+2c} \right )^{3}+\left ( \frac{c}{c+2a} \right )^{3}\geq \frac{1}{9}$

Bđt$\Leftrightarrow \left ( \frac{1}{1+\frac{2b}{a}} \right )^{3}+\left ( \frac{1}{1+\frac{2c}{b}} \right )^{3}+\left ( \frac{1}{1+\frac{2a}{c}} \right )^{3}\geq \frac{1}{9}$

Đặt $\frac{2b}{a}=\frac{2yz}{x^{2}},\frac{2c}{b}=\frac{2zx}{y^{2}},\frac{2a}{c}=\frac{2xy}{z^{2}}(x,y,z>0)$

Khi đó ta cm:

$\sum \frac{x^{6}}{(x^{2}+2yz)^{3}}\geq \frac{1}{9}$

Áp dụng bđt Xvac-xơ ta có:

$\sum \frac{x^{6}}{(x^{2}+2yz)^{3}}\geq \frac{(x^{3}+y^{3}+z^{3})^{2}}{\sum (x^{2}+2yz)^{3}}$

Ta cm: $\frac{(x^{3}+y^{3}+z^{3})^{2}}{\sum (x^{2}+2yz)^{3}}\geq \frac{1}{9}$

$\Leftrightarrow 8(x^{6}+y^{6}+z^{6})+10(x^{3}y^{3}+y^{3}z^{3}+z^{3}x^{3})\geq 6xyz(x^{3}+y^{3}+z^{3})+36x^{2}y^{2}z^{2}$

Theo AM-GM:

$2(x^{6}+x^{3}y^{3}+z^{3}x^{3})\geq 6x^{4}yz$

$2(y^{6}+y^{3}z^{3}+x^{3}y^{3})\geq 6xy^{4}z$

$2(z^{6}+z^{3}x^{3}+y^{3}z^{3})\geq 6xyz^{4}$

$6(x^{3}y^{3}+y^{3}z^{3}+z^{3}x^{3})+6(x^{6}+y^{6}+z^{6})\geq 36\sqrt[36]{(xyz)^{72}}=36x^{2}y^{2}z^{2}$

Cộng các bđt trên lại ta được đpcm


Tiếc gì một  :like nếu bạn thấy hay  :icon6:  :like  :like  :like  (Xin chân thành cảm ơn)

                                                                                                                     

                                                                                                            @};-  @};-  @};- Ôn tập phương trình tại đây !!!


#12
ineX

ineX

    Sĩ quan

  • Thành viên
  • 353 Bài viết

Một bài nữa cũng sử dụng phương pháp đổi biến:

Các số thực $x,y,z$ thỏa $xy+yz+zx=1$

Chứng minh rằng:

$P=\frac{1}{1+xy+z^{2}}+\frac{1}{1+yz+x^{2}}+\frac{1}{1+zx+y^{2}}$


"Tôi sinh ra là để thay đổi thế giới chứ không phải để thế giới thay đổi tôi" - Juliel

 

3cf67218ea144a6eb6caf571068071ff.1.gif


#13
ineX

ineX

    Sĩ quan

  • Thành viên
  • 353 Bài viết

Edit :

 

Bài 4:   Cho 3 số thực dương $x,y,z$. CMR:

 

$\frac{(x+y)^{2}}{x^{2}+y^{2}+2z^{2}}+\frac{(y+z)^{2}}{y^{2}+z^{2}+2x^{2}}+\frac{(z+x)^{2}}{z^{2}+x^{2}+2y^{2}}\leq 3$

vậy tóm lại đề bài là $\leq$ hay $\geq$ vậy?


"Tôi sinh ra là để thay đổi thế giới chứ không phải để thế giới thay đổi tôi" - Juliel

 

3cf67218ea144a6eb6caf571068071ff.1.gif


#14
NTA1907

NTA1907

    Thượng úy

  • Thành viên
  • 1014 Bài viết

vậy tóm lại đề bài là $\leq$ hay $\geq$ vậy?

Đề là $\leq$


Vũ trụ không có biên trong không gian, không có bắt đầu và kết thúc trong thời gian và chẳng có việc gì cho đấng sáng thế phải làm ở đây cả.

 


#15
nguyenduy287

nguyenduy287

    Thượng sĩ

  • Thành viên
  • 256 Bài viết

Một bài nữa cũng sử dụng phương pháp đổi biến:

Các số thực $x,y,z$ thỏa $xy+yz+zx=1$

Chứng minh rằng:

$P=\frac{1}{1+xy+z^{2}}+\frac{1}{1+yz+x^{2}}+\frac{1}{1+zx+y^{2}}$

 cmr gì vậy bạn :D


  "DÙ BẠN NGHĨ BẠN CÓ THỂ HAY BẠN KHÔNG THỂ, BẠN ĐỀU ĐÚNG "

                                                                                               -Henry Ford -

  

 

 

 

 


#16
the unknown

the unknown

    Thượng sĩ

  • Thành viên
  • 208 Bài viết

Thấy Topic khá hay mà đóng băng cũng gần một tháng rồi nên hơi buồn  :( . Mình xin đóng góp một số bài bất đẳng thức mà kĩ thuật đổi biến là một điều khá cần thiết:

Bài 5: Cho các số thực $a,b,c$ đôi một khác nhau và $k$ là số thực bất kì. Chứng minh:

$\left ( \frac{k+a}{b-c} \right )^2+\left ( \frac{k+b}{c-a} \right )^2+\left ( \frac{k+c}{a-b} \right )^2\geq 2$

Bài 6: Cho $a,b,c$ là các số thực bất kỳ. Chứng minh rằng:

$\left ( \frac{a}{a-kb} \right )^2+\left ( \frac{b}{b-kc} \right )^2+\left ( \frac{c}{c-ka} \right )^2+\frac{2(1-k^3)abc}{(a-kb)(b-kc)(c-kb)}\geq 1$

trong đó $k$ là số thực sao cho $(a-kb)(b-kc)(c-kb)\neq 0$

Bài 7: Cho $a,b,c$ là các số thực đôi một khác nhau. Chứng minh rằng khi đó với mọi $k$, ta luôn có bất đẳng thức:

$\left ( \frac{a+kb}{a-b} \right )^2+\left ( \frac{b+kc}{b-c} \right )^2+\left ( \frac{c+ka}{c-a} \right )^2\geq k^2+1$

Spoiler

Và một kết quả khá nổi tiếng của Vasile Cirtoaje:

Bài 8: Cho $x,y,z$ là các số thực dương thỏa $xyz=1$. Chứng minh rằng:

$\sum \frac{1}{x^2+x+1}\geq 1$


Bài viết đã được chỉnh sửa nội dung bởi the unknown: 04-07-2016 - 12:33

$\texttt{If you don't know where you are going, any road will get you there}$


#17
dungxibo123

dungxibo123

    Sĩ quan

  • Thành viên
  • 330 Bài viết

Thấy Topic khá hay mà đóng băng cũng gần một tháng rồi nên hơi buồn  :( . Mình xin đóng góp một số bài bất đẳng thức mà kĩ thuật đổi biến là một điều khá cần thiết:

Bài 5: Cho các số thực $a,b,c$ đôi một khác nhau và $k$ là số thực bất kì. Chứng minh:

$\left ( \frac{k+a}{b-c} \right )^2+\left ( \frac{k+b}{c-a} \right )^2+\left ( \frac{k+c}{a-b} \right )^2\geq 2$

Bài 6: Cho $a,b,c$ là các số thực bất kỳ. Chứng minh rằng:

$\left ( \frac{a}{a-kb} \right )^2+\left ( \frac{b}{b-kc} \right )^2+\left ( \frac{c}{c-ka} \right )^2+\frac{2(1-k^3)abc}{(a-kb)(b-kc)(c-kb)}\geq 1$

trong đó $k$ là số thực sao cho $(a-kb)(b-kc)(c-kb)\neq 0$

Bài 7: Cho $a,b,c$ là các số thực đôi một khác nhau. Chứng minh rằng khi đó với mọi $k$, ta luôn có bất đẳng thức:

$\left ( \frac{a+kb}{a-b} \right )^2+\left ( \frac{b+kc}{b-c} \right )^2+\left ( \frac{c+ka}{c-a} \right )^2\geq k^2+1$

Spoiler

Và một kết quả khá nổi tiếng của Vasile Cirtoaje:

Bài 8: Cho $x,y,z$ là các số thực dương thỏa $xyz=1$. Chứng minh rằng:

$\sum \frac{1}{x^2+x+1}\geq 1$

thấy không ai trả lời nên em làm bài 8

ta đổi$(x;y;z)\rightarrow (\frac{ab}{c^{2}};\frac{bc}{a^{2}};\frac{ca}{b^{2}})$

áp dụng C-S dễ dàng chứng minh được


myfb : www.facebook.com/votiendung.0805
~~~~~~~~~~~~~~~~~~~~~~~~~~~~~~~~~~~~~o0o~~~~~~~~~~~~~~~~~~~~~~~~~~~~~~~~~~~~~~
SỢ HÃI giúp ta tồn tại

NGHỊ LỰC giúp ta đứng vững

KHÁT VỌNG giúp ta tiến về phía trước

Võ Tiến Dũng  

:like  :like  :like  :like  :like 

 

 


#18
quanguefa

quanguefa

    Thiếu úy

  • Thành viên
  • 596 Bài viết

Mình xin bổ sung cho topic một số bài toán BĐT mà việc đổi biến cho ta một lời giải rất đẹp! Và những kĩ thuật đổi biến này theo mình là khá lạ với một số bạn. (mình cũng chỉ lấy từ sách ra thôi =) )

 

Bài 9 [Vasile Cirtoaje] 

CMR nếu a, b, c là những số thực không âm thì ta có: 

$2(a^2+1)(b^2+1)(c^2+1)\geq (a+1)(b+1)(c+1)(abc+1)$

Gợi ý: đặt $a=\frac{1-x}{1+x}$

 

Bài 10 [Vasile Cirtoaje]

Cho a, b, c là những số thực dương thoả mãn: $(a+b+c)(\frac{1}{a}+\frac{1}{b}+\frac{1}{c})=13$

Tìm GTNN của biểu thức: $(a^2+b^2+c^2)(\frac{1}{a^2}+\frac{1}{b^2}+\frac{1}{c^2})$

Gợi ý: đặt: $x=\sum \frac{a}{b};y=\sum \frac{b}{a}$. Chú ý mối liên hệ giữa x và y để đưa BĐT cần chứng minh về BĐT mới chỉ gồm 2 biến x, y.

 

Bài 11 [IMO 1983]

Cho a, b, c là độ dài ba cạnh một tam giác. CMR: $a^2b(a-b)+b^2c(b-c)+c^2a(c-a)\geq 0$

Gợi ý: với những bài cho a, b, c là độ dài ba cạnh một tam giác ta thường dùng phép đổi biến $x=\frac{b+c-a}{2}$... Để đưa về BĐT với các số thực dương.

 

Bài 12 [Dự bị 30/4]

Cho a, b, c là độ dài ba cạnh một tam giác. CMR: $\left | \sum \frac{a-b}{a+b} \right |< \frac{1}{16}$

Gợi ý: đổi biến tương tự như trên nhưng trước đó ta cần có một số biến đổi phù hợp.

 

Nếu không có bạn nào giải mình sẽ post lời giải lên sau =)


Bài viết đã được chỉnh sửa nội dung bởi quanguefa: 26-12-2016 - 21:08

Xem topic "Chuyên đề các bài Toán lãi suất Casio" tại đây

 

:like Visit my facebook


#19
toannguyenebolala

toannguyenebolala

    Sĩ quan

  • Thành viên
  • 432 Bài viết

Mình xin bổ sung cho topic một số bài toán BĐT mà việc đổi biến cho ta một lời giải rất đẹp! Và những kĩ thuật đổi biến này theo mình là khá lạ với một số bạn. (mình cũng chỉ lấy từ sách ra thôi =) )

 

Bài 9 [Vasile Cirtoaje] 

CMR nếu a, b, c là những số thực không âm thì ta có: 

$2(a^2+1)(b^2+1)(c^2+1)\geq (a+1)(b+1)(c+1)(abc+1)$

Gợi ý: đặt $a=\frac{1-x}{1+x}$

 

Bài 10 [Vasile Cirtoaje]

Cho a, b, c là những số thực dương thoả mãn: $(a+b+c)(\frac{1}{a}+\frac{1}{b}+\frac{1}{c})=13$

Tìm GTNN của biểu thức: $(a^2+b^2+c^2)(\frac{1}{a^2}+\frac{1}{b^2}+\frac{1}{c^2})$

Gợi ý: đặt: $x=\sum \frac{a}{b};y=\sum \frac{b}{a}$. Chú ý mối liên hệ giữa x và y để đưa BĐT cần chứng minh về BĐT mới chỉ gồm 2 biến x, y.

 

Bài 11 [IMO 1983]

Cho a, b, c là độ dài ba cạnh một tam giác. CMR: $a^2b(a-b)+b^2c(b-c)+c^2a(c-a)\geq 0$

Gợi ý: với những bài cho a, b, c là độ dài ba cạnh một tam giác ta thường dùng phép đổi biến $x=\frac{b+c-a}{2}$... Để đưa về BĐT với các số thực dương.

 

Bài 12 [Dự bị 30/4]

Cho a, b, c là độ dài ba cạnh một tam giác. CMR: $\left | \sum \frac{a-b}{a+b} \right |< \frac{1}{16}$

Gợi ý: đổi biến tương tự như trên nhưng trước đó ta cần có một số biến đổi phù hợp.

 

Nếu không có bạn nào giải mình sẽ post lời giải lên sau =)

Bài 9 đưa về chứng minh bất đẳng thức sau $(x^2+1)(y^2+1)(z^2+1)\geq \sum xy+1\Leftrightarrow (xyz)^2+\sum (x^2y^2+z^2)\geq \sum xy$ (luôn đúng) 

Vậy ta có (Q.E.D). Dấu "=" xảy ra khi và chỉ khi a=b=c=1 :D


"Đừng khóc, Alfred! Anh cần có đủ nghị lực để chết ở tuổi hai mươi"


#20
viet9a14124869

viet9a14124869

    Trung úy

  • Thành viên
  • 903 Bài viết

Topic thật sự rất hay , và rất tiếc là gần 1 năm rồi mà vẫn chưa có ai post lời giải ,mình sẽ trình bày vài bài mà mình có thể nghĩ ra :))

Bài 6: Cho $a,b,c$ là các số thực bất kỳ. Chứng minh rằng:

$\left ( \frac{a}{a-kb} \right )^2+\left ( \frac{b}{b-kc} \right )^2+\left ( \frac{c}{c-ka} \right )^2+\frac{2(1-k^3)abc}{(a-kb)(b-kc)(c-kb)}\geq 1$

trong đó $k$ là số thực sao cho $(a-kb)(b-kc)(c-kb)\neq 0$

Bài 7: Cho $a,b,c$ là các số thực đôi một khác nhau. Chứng minh rằng khi đó với mọi $k$, ta luôn có bất đẳng thức:

$\left ( \frac{a+kb}{a-b} \right )^2+\left ( \frac{b+kc}{b-c} \right )^2+\left ( \frac{c+ka}{c-a} \right )^2\geq k^2+1$

Spoiler

Bài toán 6 :

Đổi biến $x=\frac{a}{a-kb},y=\frac{b}{b-kc},z=\frac{c}{c-ka}$

Ta dễ có $x-1=\frac{kb}{a-kb}\Rightarrow (x-1)(y-1)(z-1)=\frac{k^3abc}{(a-kb)(b-kc)(c-ka)}$

Vậy đẳng thức cần chứng minh tương đương $x^2+y^2+z^2+2xyz-2(x-1)(y-1)(z-1)\geq1 \Leftrightarrow (x+y+z-1)^2\geq 0$ ( đúng )

Vậy ta có đpcm .

Bài toán 7 :

Xét trường hợp k = -1 , ta có ngay đpcm .

Nếu k khác -1 , đổi biến $x=\frac{a+kb}{a-b},y=\frac{b+kc}{b-c},z=\frac{c+ka}{c-a}$

Ta cần chứng minh : $x^2+y^2+z^2\geq k^2+1$

Mặt khác từ cách lấy x,y,z ta có :

$(x-1)(y-1)(z-1)=(x+k)(y+k)(z+k)\Leftrightarrow (k+1)(xy+yz+zx)+(k^2-1)(x+y+z)+k^3+1=0 \Leftrightarrow xy+yz+zx = -(k^2-k+1)+(1-k)(x+y+z)$

Vì $x^2+y^2+z^2-k^2-1=(x+y+z)^2-2(xy+yz+zx)-k^2-1=(x+y+z)^2+2(k-1)(x+y+z)+k^2-2k+1=(x+y+z+k-1)^2\geq 0$

Vậy ta có đpcm .

 

Bài 10 [Vasile Cirtoaje]

Cho a, b, c là những số thực dương thoả mãn: $(a+b+c)(\frac{1}{a}+\frac{1}{b}+\frac{1}{c})=13$

Tìm GTNN của biểu thức: $(a^2+b^2+c^2)(\frac{1}{a^2}+\frac{1}{b^2}+\frac{1}{c^2})$

Gợi ý: đặt: $x=\sum \frac{a}{b};y=\sum \frac{b}{a}$. Chú ý mối liên hệ giữa x và y để đưa BĐT cần chứng minh về BĐT mới chỉ gồm 2 biến x, y.

Giải bài 10 luôn :))

Đặt $x=\sum \frac{a}{b},y=\sum \frac{b}{a}\Rightarrow x+y+3=(a+b+c)(\frac{1}{a}+\frac{1}{b}+\frac{1}{c})=13\rightarrow x+y=10$

Ta có $A=(a^2+b^2+c^2)(\frac{1}{a^2}+\frac{1}{b^2}+\frac{1}{c^2})=3+\sum \frac{a^2}{b^2}+\sum\frac{b^2}{a^2}$

Mặt khác theo đánh giá $\sum \frac{a^2}{b^2}=x^2-2y,\sum \frac{b^2}{a^2}=y^2-2x$

Thì ta được $A=x^2+y^2-2x-2y+3\geq \frac{(x+y)^2}{2}-2.10+3=50-20+3=33$ ( AM-GM )

Vậy giá trị nhỏ nhất của A là 33 khi x=y=5 .


Bài viết đã được chỉnh sửa nội dung bởi viet9a14124869: 26-08-2017 - 21:26

                                                                    SÓNG BẮT ĐẦU TỪ GIÓ

                                                                    GIÓ BẮT ĐẦU TỪ ĐÂU ?

                                                                    ANH CŨNG KHÔNG BIẾT NỮA 

                                                                    KHI NÀO...? TA YÊU NHAU .





0 người đang xem chủ đề

0 thành viên, 0 khách, 0 thành viên ẩn danh